Inversi dei binomiali

Analisi, algebra lineare, topologia, gruppi, anelli, campi, ...
Rispondi
Avatar utente
Catraga
Messaggi: 302
Iscritto il: 01 gen 1970, 01:00
Località: Trieste (Univ)

Inversi dei binomiali

Messaggio da Catraga »

Dimostrare che:
$ \displaystyle \sum_{k=1}^{\infty}\frac{1}{k^2}\binom{2k}{k}^{-1}=\frac{\pi^2}{18} $
Caruccia, no?
Avatar utente
HiTLeuLeR
Messaggi: 1874
Iscritto il: 01 gen 1970, 01:00
Località: Reggio di Calabria

Messaggio da HiTLeuLeR »

Serie ipergeometriche.
Avatar utente
Catraga
Messaggi: 302
Iscritto il: 01 gen 1970, 01:00
Località: Trieste (Univ)

Messaggio da Catraga »

Ok, so cosa intendi e sono quelle per risolvere in modo standard questo tipo di problema. Ma senza di quelle?
Avatar utente
Santana
Messaggi: 72
Iscritto il: 05 feb 2006, 19:01
Contatta:

Re: Inversi dei binomiali

Messaggio da Santana »

Catraga ha scritto:Dimostrare che:
$ \displaystyle \sum_{k=1}^{\infty}\frac{1}{k^2}\binom{2k}{k}^{-1}=\frac{\pi^2}{18} $
Caruccia, no?
Indubbiamente :wink:

Sia $ B (x,y) $ la funzione beta di Eulero, dalle sue proprietà
si ricava essere $ B (n,n)=\binom{2n}{n}^{-1} $ e del resto

$ B (n,n)=\int_0^1 t^n(1-t)^n dt $

ora la tua serie

$ \sum_{k=1}^{\infty}\frac{1}{k^2}\binom{2k}{k}^{-1}=\sum_{k=1}^{\infty}\frac{1}{k^2}\int_0^1 (t-t^2)^k dt $

invertendo serie e integrale diventa

$ \int_0^1 \sum_{k=1}^{\infty} \frac{(t-t^2)^k}{k^2} dt $

credo che con un pò di cambiamenti di variabili si ottenga il risultato voluto... adesso devo andare se riesco dopo ci penso :wink:
Visita il mio nuovo sito
http://splashscreen.altervista.com/
Avatar utente
Catraga
Messaggi: 302
Iscritto il: 01 gen 1970, 01:00
Località: Trieste (Univ)

Messaggio da Catraga »

Domani mattina postero' la mia soluzione. :wink:
Intanto sotto con le soluzioni! :D
blackdie
Messaggi: 34
Iscritto il: 07 dic 2005, 18:26

Re: Inversi dei binomiali

Messaggio da blackdie »

[quote="Santana"]

$ B (n,n)=\binom{2n}{n}^{-1} $

[\quote]

Da dove viene questa uguaglianza?xke a me non sembra corretta.
Avatar utente
Santana
Messaggi: 72
Iscritto il: 05 feb 2006, 19:01
Contatta:

Re: Inversi dei binomiali

Messaggio da Santana »

blackdie ha scritto:Da dove viene questa uguaglianza?xke a me non sembra corretta.
Confondo sempre gli indici... ora non ho tempo di correggere tutto anche perchè in latex non so come inserire le lettere greche maiuscole, qui un problema simile.
Visita il mio nuovo sito
http://splashscreen.altervista.com/
EvaristeG
Site Admin
Messaggi: 4901
Iscritto il: 01 gen 1970, 01:00
Località: Roma
Contatta:

Messaggio da EvaristeG »

[completamente OT]
le lettere greche maiuscole si inseriscono mettendo la maiuscola al nome della lettera : \gamma --> \Gamma
[/completamente OT]
EvaristeG
Site Admin
Messaggi: 4901
Iscritto il: 01 gen 1970, 01:00
Località: Roma
Contatta:

Messaggio da EvaristeG »

Hmm allora : (vaghi ricordi) dovrebbe essere
$ B(m,n)=\displaystyle{\int_0^1t^{m-1}(1-t)^{n-1}dt=\frac{\Gamma(m)\Gamma(n)}{\Gamma(m+n)}} $
Ora, $ \Gamma(m)=(m-1)! $ e dunque
$ \displaystyle{B(n+1,n+1)=\frac{n!\cdot n!}{(2n + 1)!}={2n\choose n}^{-1}\frac{1}{2n+1}} $.
Quindi non è solo un problema di indici ... o mi ricordo male la beta, o c'è questo 2n+1 che avanza a denominatore in ogni pezzo della serie.
Avatar utente
Santana
Messaggi: 72
Iscritto il: 05 feb 2006, 19:01
Contatta:

Messaggio da Santana »

EvaristeG ha scritto:Hmm allora : (vaghi ricordi) dovrebbe essere
$ B(m,n)=\displaystyle{\int_0^1t^{m-1}(1-t)^{n-1}dt=\frac{\Gamma(m)\Gamma(n)}{\Gamma(m+n)}} $
Ora, $ \Gamma(m)=(m-1)! $ e dunque
$ \displaystyle{B(n+1,n+1)=\frac{n!\cdot n!}{(2n + 1)!}={2n\choose n}^{-1}\frac{1}{2n+1}} $.
Quindi non è solo un problema di indici ... o mi ricordo male la beta, o c'è questo 2n+1 che avanza a denominatore in ogni pezzo della serie.
Beh, lo so nel link è scritto proprio quello...
Visita il mio nuovo sito
http://splashscreen.altervista.com/
Avatar utente
Catraga
Messaggi: 302
Iscritto il: 01 gen 1970, 01:00
Località: Trieste (Univ)

Messaggio da Catraga »

In effetti mi rendo conto che il seguente risultato possa sembrare uscire dal nulla...
$ \displaystyle \sum_{k=1}^{\infty}\frac{1}{k^2}\binom{2k}{k}^{-1}=\left.2\left(\arctan\sqrt{\frac{t}{4t-1}}\right)^2\right|_{t=1}=\frac{\pi^2}{18} $
Ma mi piace tantissimo la soluzione con la funzione Beta! :D
Avatar utente
Santana
Messaggi: 72
Iscritto il: 05 feb 2006, 19:01
Contatta:

Messaggio da Santana »

Catraga ha scritto:In effetti mi rendo conto che il seguente risultato possa sembrare uscire dal nulla...
$ \displaystyle \sum_{k=1}^{\infty}\frac{1}{k^2}\binom{2k}{k}^{-1}=\left.2\left(\arctan\sqrt{\frac{t}{4t-1}}\right)^2\right|_{t=1}=\frac{\pi^2}{18} $
Ma mi piace tantissimo la soluzione con la funzione Beta! :D
Questa non è una dimostrazione... non ho capito, hai dimostrato o no l'identità?
Visita il mio nuovo sito
http://splashscreen.altervista.com/
EvaristeG
Site Admin
Messaggi: 4901
Iscritto il: 01 gen 1970, 01:00
Località: Roma
Contatta:

Messaggio da EvaristeG »

Onestamente non ho avuto voglia di leggere il link ... fatto sta che non è quello che c'è scritto nel tuo post :) :
Santana ha scritto:ora la tua serie
$ \sum_{k=1}^{\infty}\frac{1}{k^2}\binom{2k}{k}^{-1}=\sum_{k=1}^{\infty}\frac{1}{k^2}\int_0^1 (t-t^2)^k dt $
Mentre in realtà la serie diventa
$ \dispaystyle{\sum_{k=1}^{\infty}\frac{1}{k^2}\binom{2k}{k}^{-1}=\sum_{k=1}^{\infty}\frac{2k+1}{k^2}\int_0^1 (t-t^2)^k dt} $
Ed anche l'uguaglianza con la beta, che aveva suscitato la domanda di blackdie, non era solo affetta da un problema di indici, ma anche da un termine mancate. Per questo ho scritto la derivazione di quell'uguaglianza.
EvaristeG
Site Admin
Messaggi: 4901
Iscritto il: 01 gen 1970, 01:00
Località: Roma
Contatta:

Messaggio da EvaristeG »

Hmm ... queste somme dei binomiali centrali sono carine (o cielo, per quanto lo possa essere un integrale...). Ecco una lieve generalizzazione :
$ \displaystyle{\sum_{k=1}^\infty\frac{1}{{2k\choose k}k^n}=\sum_{k=1}^\infty\frac{1}{{2k\choose k}n!}\int_0^1\log^{n-1}\left(\frac1{x}\right)x^{k-1}dx} $
ovvero (sostituendo $ y^2=x $)
$ \displaystyle{\sum_{k=1}^\infty\frac{1}{{2k\choose k}n!}\int_0^12\log^{n-1}\left(\frac1{x}\right)y^{2k-2}(2y)dy=} $
$ \displaystyle{-\sum_{k=1}^\infty\frac{(-2)^{n-1}}{{2k\choose k}n!}\int_0^1\frac{y^{2k}}{2k}2(n-1)\frac{\log^{n-2}y}{y}dy} $ (integrando per parti derivando il log e integrando la potenza).
Da ciò, scambiando serie e integrale senza le doverose verifiche
$ \displaystyle{\frac{(-2)^{n-1}}{(n-2)!}\int_0^1\frac{\log^{n-2}y}{y}\sum_{k=1}^\infty\frac{y^{2k}}{k{2k\choose k}}dy} $
Ora, la serie si calcola con un po' di fatica (ma con le tecniche standard e un po' di conoscenza degli sviluppi di taylor) :
$ \displaystyle{\frac{(-2)^{n-1}}{(n-2)!}\int_0^1\frac{\log^{n-2}y}{y}\frac{y\arcsin\frac{y}{2}}{\sqrt{1-(\frac{y}{2})^2}}dy} $
e infine, sostituendo $ y=2\sin(\frac{\alpha}{2}) $ per levarci l'arcoseno dalle scatole, si ha
$ \displaystyle{\frac{(-2)^{n-2}}{(n-2)!}\int_0^\frac{\pi}{3}\log(2\sin(\frac{\alpha}{2}))^{n-2}dy} $
bene, il nostro caso n=2 semplifica grandemente i conteggi, restituendo proprio il valore cercato... per n>2, bisogna prendere le tabelle di quel che si chiama log-sine integral ... con un po' di pazienza, per n=4 si trova anche qui un multiplo di $ \zeta(4) $, come il caso n=2 è un multiplo di $ \zeta(2) $.
Rispondi